LSAT and Law School Admissions Forum

Get expert LSAT preparation and law school admissions advice from PowerScore Test Preparation.

User avatar
 Dave Killoran
PowerScore Staff
  • PowerScore Staff
  • Posts: 5852
  • Joined: Mar 25, 2011
|
#71213
This game is discussed in our Podcast: LSAT Podcast Episode 31: The September 2019 LSAT Logic Games Section


Complete Question Explanation
(The complete setup for this game can be found here: https://forum.powerscore.com/lsat/viewtopic.php?t=31306)

The correct answer choice is (D).

Again, try a hypothetical or two if the relationship isn't immediately clear to you (and it's not supposed to be)! Two quick ones to draw are:

  • G - P - N - S - M - H - J

    or

    N - P - M - H - S - G - J
And while neither appears to immediately prove and answer, because M and H rotate, the first hypothetical shows how M could be in case 6. Since in Could Be True questions you only need to show one instance of the answer occurring to prove it correct, that means answer choice (D) is correct.


Answer choice (A): This answer ends up putting two of J/P/S next to each other, violating the last rules.

Answer choice (B): This answer ends up putting two of J/P/S next to each other, violating the last rules.

Answer choice (C): This answer ends up putting two of J/P/S next to each other, violating the last rules.

Answer choice (D): This is the correct answer choice. This hypothetical shows how: G - P - N - S - H - M - J

Answer choice (E): This answer ends up putting two of J/P/S next to each other, violating the last rules.

Get the most out of your LSAT Prep Plus subscription.

Analyze and track your performance with our Testing and Analytics Package.